Permutation group products (23)(12)(34)=(1243)?

The name of the pictureThe name of the pictureThe name of the pictureClash Royale CLAN TAG#URR8PPP











up vote
1
down vote

favorite












I just wanted to ask why it makes sense that $(23)(12)(34)=(1243)$. Note I'm going from right to left.



I'm trying to understand the concepts trying to find a true "method" to follow in all cases. My reasoning at the moment is: $1 mapsto 2$ but since it doesn't close we must then leave the cycle open right (I'm assuming this is the case for when a cycle doesn't close unless when it returns to itself). Then we have $(12$. To proceed we consider where $2$ goes to. Then we see that $2 mapsto 3, 3 mapsto 4$. I'm getting the idea that because we see that $3$ much like a "path" to $4$ then the mapping is then $2 mapsto 4$, giving us $(124$. Then since we know $4mapsto 3$ we just close with $(1243)$. To me however it feels more accurate to see that I have $1 mapsto 2, 2mapsto 3$. Then we see that we have $(13$. Then since we went through all cycles once we then start at 2 since it hasn't closed then we go from $2 mapsto 3, 3 mapsto 4$, then we would have $(1324)$. What am I not seeing? I'm trying to imagine functions where maybe each cycle is its own function and sends to another cycle but I never seem to find anything consistent.



Thanks in advance for any of your clarifications.










share|cite|improve this question

















  • 1




    I also get $(1342)$. Could $(1243)$ be a typo?
    – Arthur
    4 hours ago











  • @Arthur Okay thanks, maybe I plugged it into WolframAlpha incorrectly.
    – dls
    4 hours ago











  • Wolfram Alpha is weird. Usually, $(2 3)(1 2)$ means you apply $(1 2)$, then you apply $(2 3)$. However, Wolfram Alpha does it so that $(2 3)(1 2)$ means you apply $(2 3)$ then $(1 2)$. Thus, they get the inverse of what it would be normally, which is $(1 2 4 3)$.
    – Noble Mushtak
    4 hours ago











  • @NobleMushtak yeah that's what I'm having some difficulty with as well. They perform the operation from left to right, which seems to be normal in some cases but not really how we're learning it in class so it a bit troublesome
    – dls
    4 hours ago






  • 2




    Unfortunately, this is something you kind of have to deal with in abstract algebra because there are two different conventions. Most people use right to left like function composition, but you always have to make sure you check what convention the article/CAS you are reading uses because some people use left to right.
    – Noble Mushtak
    4 hours ago














up vote
1
down vote

favorite












I just wanted to ask why it makes sense that $(23)(12)(34)=(1243)$. Note I'm going from right to left.



I'm trying to understand the concepts trying to find a true "method" to follow in all cases. My reasoning at the moment is: $1 mapsto 2$ but since it doesn't close we must then leave the cycle open right (I'm assuming this is the case for when a cycle doesn't close unless when it returns to itself). Then we have $(12$. To proceed we consider where $2$ goes to. Then we see that $2 mapsto 3, 3 mapsto 4$. I'm getting the idea that because we see that $3$ much like a "path" to $4$ then the mapping is then $2 mapsto 4$, giving us $(124$. Then since we know $4mapsto 3$ we just close with $(1243)$. To me however it feels more accurate to see that I have $1 mapsto 2, 2mapsto 3$. Then we see that we have $(13$. Then since we went through all cycles once we then start at 2 since it hasn't closed then we go from $2 mapsto 3, 3 mapsto 4$, then we would have $(1324)$. What am I not seeing? I'm trying to imagine functions where maybe each cycle is its own function and sends to another cycle but I never seem to find anything consistent.



Thanks in advance for any of your clarifications.










share|cite|improve this question

















  • 1




    I also get $(1342)$. Could $(1243)$ be a typo?
    – Arthur
    4 hours ago











  • @Arthur Okay thanks, maybe I plugged it into WolframAlpha incorrectly.
    – dls
    4 hours ago











  • Wolfram Alpha is weird. Usually, $(2 3)(1 2)$ means you apply $(1 2)$, then you apply $(2 3)$. However, Wolfram Alpha does it so that $(2 3)(1 2)$ means you apply $(2 3)$ then $(1 2)$. Thus, they get the inverse of what it would be normally, which is $(1 2 4 3)$.
    – Noble Mushtak
    4 hours ago











  • @NobleMushtak yeah that's what I'm having some difficulty with as well. They perform the operation from left to right, which seems to be normal in some cases but not really how we're learning it in class so it a bit troublesome
    – dls
    4 hours ago






  • 2




    Unfortunately, this is something you kind of have to deal with in abstract algebra because there are two different conventions. Most people use right to left like function composition, but you always have to make sure you check what convention the article/CAS you are reading uses because some people use left to right.
    – Noble Mushtak
    4 hours ago












up vote
1
down vote

favorite









up vote
1
down vote

favorite











I just wanted to ask why it makes sense that $(23)(12)(34)=(1243)$. Note I'm going from right to left.



I'm trying to understand the concepts trying to find a true "method" to follow in all cases. My reasoning at the moment is: $1 mapsto 2$ but since it doesn't close we must then leave the cycle open right (I'm assuming this is the case for when a cycle doesn't close unless when it returns to itself). Then we have $(12$. To proceed we consider where $2$ goes to. Then we see that $2 mapsto 3, 3 mapsto 4$. I'm getting the idea that because we see that $3$ much like a "path" to $4$ then the mapping is then $2 mapsto 4$, giving us $(124$. Then since we know $4mapsto 3$ we just close with $(1243)$. To me however it feels more accurate to see that I have $1 mapsto 2, 2mapsto 3$. Then we see that we have $(13$. Then since we went through all cycles once we then start at 2 since it hasn't closed then we go from $2 mapsto 3, 3 mapsto 4$, then we would have $(1324)$. What am I not seeing? I'm trying to imagine functions where maybe each cycle is its own function and sends to another cycle but I never seem to find anything consistent.



Thanks in advance for any of your clarifications.










share|cite|improve this question













I just wanted to ask why it makes sense that $(23)(12)(34)=(1243)$. Note I'm going from right to left.



I'm trying to understand the concepts trying to find a true "method" to follow in all cases. My reasoning at the moment is: $1 mapsto 2$ but since it doesn't close we must then leave the cycle open right (I'm assuming this is the case for when a cycle doesn't close unless when it returns to itself). Then we have $(12$. To proceed we consider where $2$ goes to. Then we see that $2 mapsto 3, 3 mapsto 4$. I'm getting the idea that because we see that $3$ much like a "path" to $4$ then the mapping is then $2 mapsto 4$, giving us $(124$. Then since we know $4mapsto 3$ we just close with $(1243)$. To me however it feels more accurate to see that I have $1 mapsto 2, 2mapsto 3$. Then we see that we have $(13$. Then since we went through all cycles once we then start at 2 since it hasn't closed then we go from $2 mapsto 3, 3 mapsto 4$, then we would have $(1324)$. What am I not seeing? I'm trying to imagine functions where maybe each cycle is its own function and sends to another cycle but I never seem to find anything consistent.



Thanks in advance for any of your clarifications.







group-theory permutations symmetric-groups permutation-cycles






share|cite|improve this question













share|cite|improve this question











share|cite|improve this question




share|cite|improve this question










asked 4 hours ago









dls

786




786







  • 1




    I also get $(1342)$. Could $(1243)$ be a typo?
    – Arthur
    4 hours ago











  • @Arthur Okay thanks, maybe I plugged it into WolframAlpha incorrectly.
    – dls
    4 hours ago











  • Wolfram Alpha is weird. Usually, $(2 3)(1 2)$ means you apply $(1 2)$, then you apply $(2 3)$. However, Wolfram Alpha does it so that $(2 3)(1 2)$ means you apply $(2 3)$ then $(1 2)$. Thus, they get the inverse of what it would be normally, which is $(1 2 4 3)$.
    – Noble Mushtak
    4 hours ago











  • @NobleMushtak yeah that's what I'm having some difficulty with as well. They perform the operation from left to right, which seems to be normal in some cases but not really how we're learning it in class so it a bit troublesome
    – dls
    4 hours ago






  • 2




    Unfortunately, this is something you kind of have to deal with in abstract algebra because there are two different conventions. Most people use right to left like function composition, but you always have to make sure you check what convention the article/CAS you are reading uses because some people use left to right.
    – Noble Mushtak
    4 hours ago












  • 1




    I also get $(1342)$. Could $(1243)$ be a typo?
    – Arthur
    4 hours ago











  • @Arthur Okay thanks, maybe I plugged it into WolframAlpha incorrectly.
    – dls
    4 hours ago











  • Wolfram Alpha is weird. Usually, $(2 3)(1 2)$ means you apply $(1 2)$, then you apply $(2 3)$. However, Wolfram Alpha does it so that $(2 3)(1 2)$ means you apply $(2 3)$ then $(1 2)$. Thus, they get the inverse of what it would be normally, which is $(1 2 4 3)$.
    – Noble Mushtak
    4 hours ago











  • @NobleMushtak yeah that's what I'm having some difficulty with as well. They perform the operation from left to right, which seems to be normal in some cases but not really how we're learning it in class so it a bit troublesome
    – dls
    4 hours ago






  • 2




    Unfortunately, this is something you kind of have to deal with in abstract algebra because there are two different conventions. Most people use right to left like function composition, but you always have to make sure you check what convention the article/CAS you are reading uses because some people use left to right.
    – Noble Mushtak
    4 hours ago







1




1




I also get $(1342)$. Could $(1243)$ be a typo?
– Arthur
4 hours ago





I also get $(1342)$. Could $(1243)$ be a typo?
– Arthur
4 hours ago













@Arthur Okay thanks, maybe I plugged it into WolframAlpha incorrectly.
– dls
4 hours ago





@Arthur Okay thanks, maybe I plugged it into WolframAlpha incorrectly.
– dls
4 hours ago













Wolfram Alpha is weird. Usually, $(2 3)(1 2)$ means you apply $(1 2)$, then you apply $(2 3)$. However, Wolfram Alpha does it so that $(2 3)(1 2)$ means you apply $(2 3)$ then $(1 2)$. Thus, they get the inverse of what it would be normally, which is $(1 2 4 3)$.
– Noble Mushtak
4 hours ago





Wolfram Alpha is weird. Usually, $(2 3)(1 2)$ means you apply $(1 2)$, then you apply $(2 3)$. However, Wolfram Alpha does it so that $(2 3)(1 2)$ means you apply $(2 3)$ then $(1 2)$. Thus, they get the inverse of what it would be normally, which is $(1 2 4 3)$.
– Noble Mushtak
4 hours ago













@NobleMushtak yeah that's what I'm having some difficulty with as well. They perform the operation from left to right, which seems to be normal in some cases but not really how we're learning it in class so it a bit troublesome
– dls
4 hours ago




@NobleMushtak yeah that's what I'm having some difficulty with as well. They perform the operation from left to right, which seems to be normal in some cases but not really how we're learning it in class so it a bit troublesome
– dls
4 hours ago




2




2




Unfortunately, this is something you kind of have to deal with in abstract algebra because there are two different conventions. Most people use right to left like function composition, but you always have to make sure you check what convention the article/CAS you are reading uses because some people use left to right.
– Noble Mushtak
4 hours ago




Unfortunately, this is something you kind of have to deal with in abstract algebra because there are two different conventions. Most people use right to left like function composition, but you always have to make sure you check what convention the article/CAS you are reading uses because some people use left to right.
– Noble Mushtak
4 hours ago










4 Answers
4






active

oldest

votes

















up vote
2
down vote



accepted










I would also get $(1342)$ if I were to calculate that product, following basically the same argument you did.



If you got $(1243)$ from WolframAlpha, note what it says in the input interpretation field:




the product $sigmatau$ represents the permutation $tau(sigma(cdot))$




meaning they apply permutations from left to right (first $sigma$, then $tau$). In that light, their calculations are entirely correct, but they do not get the same answer as you do because they interpret the product of permutations differently.



At least they are nice enough to tell you what convention they use.






share|cite|improve this answer






















  • Thanks for that clarification! I guess I'm tired of computing these cycle products and messed up plugging it in. Wish I knew more programming to just compute these for me haha
    – dls
    4 hours ago


















up vote
1
down vote













For me, cycle notation is always confusing when multiplying, so I always try to write the whole permutation out. I will also go right to left, so we start with $(34)$:



$$left(beginmatrix 1 & 2 & 3 & 4 \ 1 & 2 & 4 & 3endmatrixright)$$



Then, we apply $(12)$:



$$left(beginmatrix 1 & 2 & 3 & 4 \ 2 & 1 & 4 & 3endmatrixright)$$



Finally, we apply $(23)$:



$$left(beginmatrix 1 & 2 & 3 & 4 \ 3 & 1 & 4 & 2endmatrixright)$$



Thus, $1 to 3 to 4 to 2 to 1$, so we get the cycle $(1342)$. This is the same answer you got, so I hope this alternative method gives you more confidence that your reasoning is correct!






share|cite|improve this answer




















  • Thanks for the reminder, I do feel more confident with the array form as well haha
    – dls
    4 hours ago

















up vote
1
down vote













You want to clarify $$(23)(12)(34)=(1243)$$



We need to move from left to right through these permutations.



Well, let us start with $1$



$1$ does not move under $(2,3)$ but it goes to $2$ under $(1,2)$ and does not move under $(3,4)$.



As the result, we have $$ 1to 2$$
Now let us see what happens to $2$



Well $2$ goes to $3$ under $(2,3)$ and $3$ does not move under $(1,2)$ and maps to $4$ under $(3,4)$
As the result $$ 2to 4$$



Let us see what happens to $4$



Well , $4$ does not move under $(2,3)$ and $(1,2)$ but maps to $3$ under $(3,4)$.



As the result $$ 4to 3$$



Let us see what happens to $3$



Well , $3$ moves to $2$ under $(2,3)$ and $2$ moves to
$1$ under $(1,2)$ and $1$ does not move under $(3,4)$.



As the result $$ 3to 1$$



The final result is $$ (23)(12)(34)=(1243)$$



Which is what we want to prove.






share|cite|improve this answer




















  • Thanks! I was doing this from right to left and it is nice to be shown the other way too :)
    – dls
    3 hours ago










  • @dls I do not know why they do that but they do it anyway. Thanks for your comment.
    – Mohammad Riazi-Kermani
    2 hours ago

















up vote
-1
down vote













Just plug in numbers and see where they end up when you finish going through (23)(12)(34) a single time.



Plug in 1: It first gets sent to 2 by (12), then that 2 gets sent to 3 by (23), so we end at 3. Hence, 1 -> 3, so we start with (13 . [sic]



Plug in 3: It goes to 4 via (34), but the 4 doesn’t get sent anywhere after that (34). Hence, 3 -> 4, and we now have (134 . [sic]



Plug in 4: It goes to 3 via (34), and that 3 goes to 2 by (23). Hence, 4 -> 2, and we have (1342 . [sic]



Plug in 2: It goes to 1 via (12), and that 1 doesn’t go anywhere else. Hence, 2 -> 1. Since 1 is the first entry of (1342 , this means we have closed the loop—literally, we close the loop and get (1342) (by adding a parenthesis :P).






share|cite|improve this answer
















  • 1




    That's exactly what dls has already done, if you read the question post.
    – Arthur
    4 hours ago











Your Answer




StackExchange.ifUsing("editor", function ()
return StackExchange.using("mathjaxEditing", function ()
StackExchange.MarkdownEditor.creationCallbacks.add(function (editor, postfix)
StackExchange.mathjaxEditing.prepareWmdForMathJax(editor, postfix, [["$", "$"], ["\\(","\\)"]]);
);
);
, "mathjax-editing");

StackExchange.ready(function()
var channelOptions =
tags: "".split(" "),
id: "69"
;
initTagRenderer("".split(" "), "".split(" "), channelOptions);

StackExchange.using("externalEditor", function()
// Have to fire editor after snippets, if snippets enabled
if (StackExchange.settings.snippets.snippetsEnabled)
StackExchange.using("snippets", function()
createEditor();
);

else
createEditor();

);

function createEditor()
StackExchange.prepareEditor(
heartbeatType: 'answer',
convertImagesToLinks: true,
noModals: false,
showLowRepImageUploadWarning: true,
reputationToPostImages: 10,
bindNavPrevention: true,
postfix: "",
noCode: true, onDemand: true,
discardSelector: ".discard-answer"
,immediatelyShowMarkdownHelp:true
);



);













 

draft saved


draft discarded


















StackExchange.ready(
function ()
StackExchange.openid.initPostLogin('.new-post-login', 'https%3a%2f%2fmath.stackexchange.com%2fquestions%2f2963845%2fpermutation-group-products-231234-1243%23new-answer', 'question_page');

);

Post as a guest






























4 Answers
4






active

oldest

votes








4 Answers
4






active

oldest

votes









active

oldest

votes






active

oldest

votes








up vote
2
down vote



accepted










I would also get $(1342)$ if I were to calculate that product, following basically the same argument you did.



If you got $(1243)$ from WolframAlpha, note what it says in the input interpretation field:




the product $sigmatau$ represents the permutation $tau(sigma(cdot))$




meaning they apply permutations from left to right (first $sigma$, then $tau$). In that light, their calculations are entirely correct, but they do not get the same answer as you do because they interpret the product of permutations differently.



At least they are nice enough to tell you what convention they use.






share|cite|improve this answer






















  • Thanks for that clarification! I guess I'm tired of computing these cycle products and messed up plugging it in. Wish I knew more programming to just compute these for me haha
    – dls
    4 hours ago















up vote
2
down vote



accepted










I would also get $(1342)$ if I were to calculate that product, following basically the same argument you did.



If you got $(1243)$ from WolframAlpha, note what it says in the input interpretation field:




the product $sigmatau$ represents the permutation $tau(sigma(cdot))$




meaning they apply permutations from left to right (first $sigma$, then $tau$). In that light, their calculations are entirely correct, but they do not get the same answer as you do because they interpret the product of permutations differently.



At least they are nice enough to tell you what convention they use.






share|cite|improve this answer






















  • Thanks for that clarification! I guess I'm tired of computing these cycle products and messed up plugging it in. Wish I knew more programming to just compute these for me haha
    – dls
    4 hours ago













up vote
2
down vote



accepted







up vote
2
down vote



accepted






I would also get $(1342)$ if I were to calculate that product, following basically the same argument you did.



If you got $(1243)$ from WolframAlpha, note what it says in the input interpretation field:




the product $sigmatau$ represents the permutation $tau(sigma(cdot))$




meaning they apply permutations from left to right (first $sigma$, then $tau$). In that light, their calculations are entirely correct, but they do not get the same answer as you do because they interpret the product of permutations differently.



At least they are nice enough to tell you what convention they use.






share|cite|improve this answer














I would also get $(1342)$ if I were to calculate that product, following basically the same argument you did.



If you got $(1243)$ from WolframAlpha, note what it says in the input interpretation field:




the product $sigmatau$ represents the permutation $tau(sigma(cdot))$




meaning they apply permutations from left to right (first $sigma$, then $tau$). In that light, their calculations are entirely correct, but they do not get the same answer as you do because they interpret the product of permutations differently.



At least they are nice enough to tell you what convention they use.







share|cite|improve this answer














share|cite|improve this answer



share|cite|improve this answer








edited 4 hours ago

























answered 4 hours ago









Arthur

105k799182




105k799182











  • Thanks for that clarification! I guess I'm tired of computing these cycle products and messed up plugging it in. Wish I knew more programming to just compute these for me haha
    – dls
    4 hours ago

















  • Thanks for that clarification! I guess I'm tired of computing these cycle products and messed up plugging it in. Wish I knew more programming to just compute these for me haha
    – dls
    4 hours ago
















Thanks for that clarification! I guess I'm tired of computing these cycle products and messed up plugging it in. Wish I knew more programming to just compute these for me haha
– dls
4 hours ago





Thanks for that clarification! I guess I'm tired of computing these cycle products and messed up plugging it in. Wish I knew more programming to just compute these for me haha
– dls
4 hours ago











up vote
1
down vote













For me, cycle notation is always confusing when multiplying, so I always try to write the whole permutation out. I will also go right to left, so we start with $(34)$:



$$left(beginmatrix 1 & 2 & 3 & 4 \ 1 & 2 & 4 & 3endmatrixright)$$



Then, we apply $(12)$:



$$left(beginmatrix 1 & 2 & 3 & 4 \ 2 & 1 & 4 & 3endmatrixright)$$



Finally, we apply $(23)$:



$$left(beginmatrix 1 & 2 & 3 & 4 \ 3 & 1 & 4 & 2endmatrixright)$$



Thus, $1 to 3 to 4 to 2 to 1$, so we get the cycle $(1342)$. This is the same answer you got, so I hope this alternative method gives you more confidence that your reasoning is correct!






share|cite|improve this answer




















  • Thanks for the reminder, I do feel more confident with the array form as well haha
    – dls
    4 hours ago














up vote
1
down vote













For me, cycle notation is always confusing when multiplying, so I always try to write the whole permutation out. I will also go right to left, so we start with $(34)$:



$$left(beginmatrix 1 & 2 & 3 & 4 \ 1 & 2 & 4 & 3endmatrixright)$$



Then, we apply $(12)$:



$$left(beginmatrix 1 & 2 & 3 & 4 \ 2 & 1 & 4 & 3endmatrixright)$$



Finally, we apply $(23)$:



$$left(beginmatrix 1 & 2 & 3 & 4 \ 3 & 1 & 4 & 2endmatrixright)$$



Thus, $1 to 3 to 4 to 2 to 1$, so we get the cycle $(1342)$. This is the same answer you got, so I hope this alternative method gives you more confidence that your reasoning is correct!






share|cite|improve this answer




















  • Thanks for the reminder, I do feel more confident with the array form as well haha
    – dls
    4 hours ago












up vote
1
down vote










up vote
1
down vote









For me, cycle notation is always confusing when multiplying, so I always try to write the whole permutation out. I will also go right to left, so we start with $(34)$:



$$left(beginmatrix 1 & 2 & 3 & 4 \ 1 & 2 & 4 & 3endmatrixright)$$



Then, we apply $(12)$:



$$left(beginmatrix 1 & 2 & 3 & 4 \ 2 & 1 & 4 & 3endmatrixright)$$



Finally, we apply $(23)$:



$$left(beginmatrix 1 & 2 & 3 & 4 \ 3 & 1 & 4 & 2endmatrixright)$$



Thus, $1 to 3 to 4 to 2 to 1$, so we get the cycle $(1342)$. This is the same answer you got, so I hope this alternative method gives you more confidence that your reasoning is correct!






share|cite|improve this answer












For me, cycle notation is always confusing when multiplying, so I always try to write the whole permutation out. I will also go right to left, so we start with $(34)$:



$$left(beginmatrix 1 & 2 & 3 & 4 \ 1 & 2 & 4 & 3endmatrixright)$$



Then, we apply $(12)$:



$$left(beginmatrix 1 & 2 & 3 & 4 \ 2 & 1 & 4 & 3endmatrixright)$$



Finally, we apply $(23)$:



$$left(beginmatrix 1 & 2 & 3 & 4 \ 3 & 1 & 4 & 2endmatrixright)$$



Thus, $1 to 3 to 4 to 2 to 1$, so we get the cycle $(1342)$. This is the same answer you got, so I hope this alternative method gives you more confidence that your reasoning is correct!







share|cite|improve this answer












share|cite|improve this answer



share|cite|improve this answer










answered 4 hours ago









Noble Mushtak

12.6k1632




12.6k1632











  • Thanks for the reminder, I do feel more confident with the array form as well haha
    – dls
    4 hours ago
















  • Thanks for the reminder, I do feel more confident with the array form as well haha
    – dls
    4 hours ago















Thanks for the reminder, I do feel more confident with the array form as well haha
– dls
4 hours ago




Thanks for the reminder, I do feel more confident with the array form as well haha
– dls
4 hours ago










up vote
1
down vote













You want to clarify $$(23)(12)(34)=(1243)$$



We need to move from left to right through these permutations.



Well, let us start with $1$



$1$ does not move under $(2,3)$ but it goes to $2$ under $(1,2)$ and does not move under $(3,4)$.



As the result, we have $$ 1to 2$$
Now let us see what happens to $2$



Well $2$ goes to $3$ under $(2,3)$ and $3$ does not move under $(1,2)$ and maps to $4$ under $(3,4)$
As the result $$ 2to 4$$



Let us see what happens to $4$



Well , $4$ does not move under $(2,3)$ and $(1,2)$ but maps to $3$ under $(3,4)$.



As the result $$ 4to 3$$



Let us see what happens to $3$



Well , $3$ moves to $2$ under $(2,3)$ and $2$ moves to
$1$ under $(1,2)$ and $1$ does not move under $(3,4)$.



As the result $$ 3to 1$$



The final result is $$ (23)(12)(34)=(1243)$$



Which is what we want to prove.






share|cite|improve this answer




















  • Thanks! I was doing this from right to left and it is nice to be shown the other way too :)
    – dls
    3 hours ago










  • @dls I do not know why they do that but they do it anyway. Thanks for your comment.
    – Mohammad Riazi-Kermani
    2 hours ago














up vote
1
down vote













You want to clarify $$(23)(12)(34)=(1243)$$



We need to move from left to right through these permutations.



Well, let us start with $1$



$1$ does not move under $(2,3)$ but it goes to $2$ under $(1,2)$ and does not move under $(3,4)$.



As the result, we have $$ 1to 2$$
Now let us see what happens to $2$



Well $2$ goes to $3$ under $(2,3)$ and $3$ does not move under $(1,2)$ and maps to $4$ under $(3,4)$
As the result $$ 2to 4$$



Let us see what happens to $4$



Well , $4$ does not move under $(2,3)$ and $(1,2)$ but maps to $3$ under $(3,4)$.



As the result $$ 4to 3$$



Let us see what happens to $3$



Well , $3$ moves to $2$ under $(2,3)$ and $2$ moves to
$1$ under $(1,2)$ and $1$ does not move under $(3,4)$.



As the result $$ 3to 1$$



The final result is $$ (23)(12)(34)=(1243)$$



Which is what we want to prove.






share|cite|improve this answer




















  • Thanks! I was doing this from right to left and it is nice to be shown the other way too :)
    – dls
    3 hours ago










  • @dls I do not know why they do that but they do it anyway. Thanks for your comment.
    – Mohammad Riazi-Kermani
    2 hours ago












up vote
1
down vote










up vote
1
down vote









You want to clarify $$(23)(12)(34)=(1243)$$



We need to move from left to right through these permutations.



Well, let us start with $1$



$1$ does not move under $(2,3)$ but it goes to $2$ under $(1,2)$ and does not move under $(3,4)$.



As the result, we have $$ 1to 2$$
Now let us see what happens to $2$



Well $2$ goes to $3$ under $(2,3)$ and $3$ does not move under $(1,2)$ and maps to $4$ under $(3,4)$
As the result $$ 2to 4$$



Let us see what happens to $4$



Well , $4$ does not move under $(2,3)$ and $(1,2)$ but maps to $3$ under $(3,4)$.



As the result $$ 4to 3$$



Let us see what happens to $3$



Well , $3$ moves to $2$ under $(2,3)$ and $2$ moves to
$1$ under $(1,2)$ and $1$ does not move under $(3,4)$.



As the result $$ 3to 1$$



The final result is $$ (23)(12)(34)=(1243)$$



Which is what we want to prove.






share|cite|improve this answer












You want to clarify $$(23)(12)(34)=(1243)$$



We need to move from left to right through these permutations.



Well, let us start with $1$



$1$ does not move under $(2,3)$ but it goes to $2$ under $(1,2)$ and does not move under $(3,4)$.



As the result, we have $$ 1to 2$$
Now let us see what happens to $2$



Well $2$ goes to $3$ under $(2,3)$ and $3$ does not move under $(1,2)$ and maps to $4$ under $(3,4)$
As the result $$ 2to 4$$



Let us see what happens to $4$



Well , $4$ does not move under $(2,3)$ and $(1,2)$ but maps to $3$ under $(3,4)$.



As the result $$ 4to 3$$



Let us see what happens to $3$



Well , $3$ moves to $2$ under $(2,3)$ and $2$ moves to
$1$ under $(1,2)$ and $1$ does not move under $(3,4)$.



As the result $$ 3to 1$$



The final result is $$ (23)(12)(34)=(1243)$$



Which is what we want to prove.







share|cite|improve this answer












share|cite|improve this answer



share|cite|improve this answer










answered 4 hours ago









Mohammad Riazi-Kermani

35.5k41855




35.5k41855











  • Thanks! I was doing this from right to left and it is nice to be shown the other way too :)
    – dls
    3 hours ago










  • @dls I do not know why they do that but they do it anyway. Thanks for your comment.
    – Mohammad Riazi-Kermani
    2 hours ago
















  • Thanks! I was doing this from right to left and it is nice to be shown the other way too :)
    – dls
    3 hours ago










  • @dls I do not know why they do that but they do it anyway. Thanks for your comment.
    – Mohammad Riazi-Kermani
    2 hours ago















Thanks! I was doing this from right to left and it is nice to be shown the other way too :)
– dls
3 hours ago




Thanks! I was doing this from right to left and it is nice to be shown the other way too :)
– dls
3 hours ago












@dls I do not know why they do that but they do it anyway. Thanks for your comment.
– Mohammad Riazi-Kermani
2 hours ago




@dls I do not know why they do that but they do it anyway. Thanks for your comment.
– Mohammad Riazi-Kermani
2 hours ago










up vote
-1
down vote













Just plug in numbers and see where they end up when you finish going through (23)(12)(34) a single time.



Plug in 1: It first gets sent to 2 by (12), then that 2 gets sent to 3 by (23), so we end at 3. Hence, 1 -> 3, so we start with (13 . [sic]



Plug in 3: It goes to 4 via (34), but the 4 doesn’t get sent anywhere after that (34). Hence, 3 -> 4, and we now have (134 . [sic]



Plug in 4: It goes to 3 via (34), and that 3 goes to 2 by (23). Hence, 4 -> 2, and we have (1342 . [sic]



Plug in 2: It goes to 1 via (12), and that 1 doesn’t go anywhere else. Hence, 2 -> 1. Since 1 is the first entry of (1342 , this means we have closed the loop—literally, we close the loop and get (1342) (by adding a parenthesis :P).






share|cite|improve this answer
















  • 1




    That's exactly what dls has already done, if you read the question post.
    – Arthur
    4 hours ago















up vote
-1
down vote













Just plug in numbers and see where they end up when you finish going through (23)(12)(34) a single time.



Plug in 1: It first gets sent to 2 by (12), then that 2 gets sent to 3 by (23), so we end at 3. Hence, 1 -> 3, so we start with (13 . [sic]



Plug in 3: It goes to 4 via (34), but the 4 doesn’t get sent anywhere after that (34). Hence, 3 -> 4, and we now have (134 . [sic]



Plug in 4: It goes to 3 via (34), and that 3 goes to 2 by (23). Hence, 4 -> 2, and we have (1342 . [sic]



Plug in 2: It goes to 1 via (12), and that 1 doesn’t go anywhere else. Hence, 2 -> 1. Since 1 is the first entry of (1342 , this means we have closed the loop—literally, we close the loop and get (1342) (by adding a parenthesis :P).






share|cite|improve this answer
















  • 1




    That's exactly what dls has already done, if you read the question post.
    – Arthur
    4 hours ago













up vote
-1
down vote










up vote
-1
down vote









Just plug in numbers and see where they end up when you finish going through (23)(12)(34) a single time.



Plug in 1: It first gets sent to 2 by (12), then that 2 gets sent to 3 by (23), so we end at 3. Hence, 1 -> 3, so we start with (13 . [sic]



Plug in 3: It goes to 4 via (34), but the 4 doesn’t get sent anywhere after that (34). Hence, 3 -> 4, and we now have (134 . [sic]



Plug in 4: It goes to 3 via (34), and that 3 goes to 2 by (23). Hence, 4 -> 2, and we have (1342 . [sic]



Plug in 2: It goes to 1 via (12), and that 1 doesn’t go anywhere else. Hence, 2 -> 1. Since 1 is the first entry of (1342 , this means we have closed the loop—literally, we close the loop and get (1342) (by adding a parenthesis :P).






share|cite|improve this answer












Just plug in numbers and see where they end up when you finish going through (23)(12)(34) a single time.



Plug in 1: It first gets sent to 2 by (12), then that 2 gets sent to 3 by (23), so we end at 3. Hence, 1 -> 3, so we start with (13 . [sic]



Plug in 3: It goes to 4 via (34), but the 4 doesn’t get sent anywhere after that (34). Hence, 3 -> 4, and we now have (134 . [sic]



Plug in 4: It goes to 3 via (34), and that 3 goes to 2 by (23). Hence, 4 -> 2, and we have (1342 . [sic]



Plug in 2: It goes to 1 via (12), and that 1 doesn’t go anywhere else. Hence, 2 -> 1. Since 1 is the first entry of (1342 , this means we have closed the loop—literally, we close the loop and get (1342) (by adding a parenthesis :P).







share|cite|improve this answer












share|cite|improve this answer



share|cite|improve this answer










answered 4 hours ago









Junhugie

644




644







  • 1




    That's exactly what dls has already done, if you read the question post.
    – Arthur
    4 hours ago













  • 1




    That's exactly what dls has already done, if you read the question post.
    – Arthur
    4 hours ago








1




1




That's exactly what dls has already done, if you read the question post.
– Arthur
4 hours ago





That's exactly what dls has already done, if you read the question post.
– Arthur
4 hours ago


















 

draft saved


draft discarded















































 


draft saved


draft discarded














StackExchange.ready(
function ()
StackExchange.openid.initPostLogin('.new-post-login', 'https%3a%2f%2fmath.stackexchange.com%2fquestions%2f2963845%2fpermutation-group-products-231234-1243%23new-answer', 'question_page');

);

Post as a guest













































































Comments

Popular posts from this blog

Long meetings (6-7 hours a day): Being “babysat” by supervisor

Is the Concept of Multiple Fantasy Races Scientifically Flawed? [closed]

Confectionery